Multivariate limit of $|x|^$How do I find the limit of $fracxysqrtx^2 + xy + y^2$ as x and y approach zero?Proving inexistence of limitMultivariable limit of rational functionCalculate double limit of $x^2sinfrac1xy$Difficulty proving multivariate limit involving sin^2(x) does not existProof that the limit exists using polar coordinatesProving Multivariate Limit with Squeeze TheoremTwo-variable limit of $lim_(x,y)to(0,0)fracsin(x^4+y^4)x^2+y^2$Calculate if this Multivariable Limit existhow to prove limit exists or not?

I'm in charge of equipment buying but no one's ever happy with what I choose. How to fix this?

Generic lambda vs generic function give different behaviour

Confused about a passage in Harry Potter y la piedra filosofal

What would happen if the UK refused to take part in EU Parliamentary elections?

What's the purpose of "true" in bash "if sudo true; then"

Can I use my Chinese passport to enter China after I acquired another citizenship?

Can somebody explain Brexit in a few child-proof sentences?

Is there a problem with hiding "forgot password" until it's needed?

Valid Badminton Score?

Increase performance creating Mandelbrot set in python

Why "be dealt cards" rather than "be dealing cards"?

when is out of tune ok?

Why does John Bercow say “unlock” after reading out the results of a vote?

Is HostGator storing my password in plaintext?

Using parameter substitution on a Bash array

Have I saved too much for retirement so far?

Greatest common substring

Can I Retrieve Email Addresses from BCC?

Efficiently merge handle parallel feature branches in SFDX

How do I rename a LINUX host without needing to reboot for the rename to take effect?

Failed to fetch jessie backports repository

Implement the Thanos sorting algorithm

How do I define a right arrow with bar in LaTeX?

Is it okay / does it make sense for another player to join a running game of Munchkin?



Multivariate limit of $|x|^y$


How do I find the limit of $fracxysqrtx^2 + xy + y^2$ as x and y approach zero?Proving inexistence of limitMultivariable limit of rational functionCalculate double limit of $x^2sinfrac1xy$Difficulty proving multivariate limit involving sin^2(x) does not existProof that the limit exists using polar coordinatesProving Multivariate Limit with Squeeze TheoremTwo-variable limit of $lim_(x,y)to(0,0)fracsin(x^4+y^4)x^2+y^2$Calculate if this Multivariable Limit existhow to prove limit exists or not?













1












$begingroup$


I'm trying to prove that the limit:



$lim_(x,y)to (0,0)|x|^y=1$



I started with
$lim_(x,y)to (0,0)^y=lim_(x,y)to (0,0)e^=e^lim_(x,y)to (0,0)$



And then tried substituting $x=rsintheta$ and $y=rcostheta$ to get:



$e^lim_rto 0rcostheta$



However, as I'm trying to use the squeeze theorem, I can't figure out a lower bound that would work on this. Any suggestions? Does the limit even exist?










share|cite|improve this question











$endgroup$











  • $begingroup$
    See my answer to Multivariable limits.
    $endgroup$
    – Dave L. Renfro
    Mar 17 at 12:43
















1












$begingroup$


I'm trying to prove that the limit:



$lim_(x,y)to (0,0)|x|^y=1$



I started with
$lim_(x,y)to (0,0)^y=lim_(x,y)to (0,0)e^=e^lim_(x,y)to (0,0)$



And then tried substituting $x=rsintheta$ and $y=rcostheta$ to get:



$e^lim_rto 0rcostheta$



However, as I'm trying to use the squeeze theorem, I can't figure out a lower bound that would work on this. Any suggestions? Does the limit even exist?










share|cite|improve this question











$endgroup$











  • $begingroup$
    See my answer to Multivariable limits.
    $endgroup$
    – Dave L. Renfro
    Mar 17 at 12:43














1












1








1


1



$begingroup$


I'm trying to prove that the limit:



$lim_(x,y)to (0,0)|x|^y=1$



I started with
$lim_(x,y)to (0,0)^y=lim_(x,y)to (0,0)e^=e^lim_(x,y)to (0,0)$



And then tried substituting $x=rsintheta$ and $y=rcostheta$ to get:



$e^lim_rto 0rcostheta$



However, as I'm trying to use the squeeze theorem, I can't figure out a lower bound that would work on this. Any suggestions? Does the limit even exist?










share|cite|improve this question











$endgroup$




I'm trying to prove that the limit:



$lim_(x,y)to (0,0)|x|^y=1$



I started with
$lim_(x,y)to (0,0)^y=lim_(x,y)to (0,0)e^=e^lim_(x,y)to (0,0)$



And then tried substituting $x=rsintheta$ and $y=rcostheta$ to get:



$e^lim_rto 0rcostheta$



However, as I'm trying to use the squeeze theorem, I can't figure out a lower bound that would work on this. Any suggestions? Does the limit even exist?







limits multivariable-calculus






share|cite|improve this question















share|cite|improve this question













share|cite|improve this question




share|cite|improve this question








edited Mar 17 at 13:17









YuiTo Cheng

2,1362837




2,1362837










asked Mar 17 at 12:35









user653788user653788

102




102











  • $begingroup$
    See my answer to Multivariable limits.
    $endgroup$
    – Dave L. Renfro
    Mar 17 at 12:43

















  • $begingroup$
    See my answer to Multivariable limits.
    $endgroup$
    – Dave L. Renfro
    Mar 17 at 12:43
















$begingroup$
See my answer to Multivariable limits.
$endgroup$
– Dave L. Renfro
Mar 17 at 12:43





$begingroup$
See my answer to Multivariable limits.
$endgroup$
– Dave L. Renfro
Mar 17 at 12:43











2 Answers
2






active

oldest

votes


















2












$begingroup$

This limit doesn't exist. Consider two different paths $y=0$ and $x=0$. One leads to $0$-limit and the other to $1$-limit. The figure below shows why:
enter image description here






share|cite|improve this answer









$endgroup$




















    2












    $begingroup$

    The limit does not exist: When approaching $(0,0)$ along the $x$ axis we have $|x|^y=|x|^0=1$, while when approaching along the $y$ axis we have $|x|^y=0^y=0$.






    share|cite|improve this answer











    $endgroup$












      Your Answer





      StackExchange.ifUsing("editor", function ()
      return StackExchange.using("mathjaxEditing", function ()
      StackExchange.MarkdownEditor.creationCallbacks.add(function (editor, postfix)
      StackExchange.mathjaxEditing.prepareWmdForMathJax(editor, postfix, [["$", "$"], ["\\(","\\)"]]);
      );
      );
      , "mathjax-editing");

      StackExchange.ready(function()
      var channelOptions =
      tags: "".split(" "),
      id: "69"
      ;
      initTagRenderer("".split(" "), "".split(" "), channelOptions);

      StackExchange.using("externalEditor", function()
      // Have to fire editor after snippets, if snippets enabled
      if (StackExchange.settings.snippets.snippetsEnabled)
      StackExchange.using("snippets", function()
      createEditor();
      );

      else
      createEditor();

      );

      function createEditor()
      StackExchange.prepareEditor(
      heartbeatType: 'answer',
      autoActivateHeartbeat: false,
      convertImagesToLinks: true,
      noModals: true,
      showLowRepImageUploadWarning: true,
      reputationToPostImages: 10,
      bindNavPrevention: true,
      postfix: "",
      imageUploader:
      brandingHtml: "Powered by u003ca class="icon-imgur-white" href="https://imgur.com/"u003eu003c/au003e",
      contentPolicyHtml: "User contributions licensed under u003ca href="https://creativecommons.org/licenses/by-sa/3.0/"u003ecc by-sa 3.0 with attribution requiredu003c/au003e u003ca href="https://stackoverflow.com/legal/content-policy"u003e(content policy)u003c/au003e",
      allowUrls: true
      ,
      noCode: true, onDemand: true,
      discardSelector: ".discard-answer"
      ,immediatelyShowMarkdownHelp:true
      );



      );













      draft saved

      draft discarded


















      StackExchange.ready(
      function ()
      StackExchange.openid.initPostLogin('.new-post-login', 'https%3a%2f%2fmath.stackexchange.com%2fquestions%2f3151488%2fmultivariate-limit-of-xy%23new-answer', 'question_page');

      );

      Post as a guest















      Required, but never shown

























      2 Answers
      2






      active

      oldest

      votes








      2 Answers
      2






      active

      oldest

      votes









      active

      oldest

      votes






      active

      oldest

      votes









      2












      $begingroup$

      This limit doesn't exist. Consider two different paths $y=0$ and $x=0$. One leads to $0$-limit and the other to $1$-limit. The figure below shows why:
      enter image description here






      share|cite|improve this answer









      $endgroup$

















        2












        $begingroup$

        This limit doesn't exist. Consider two different paths $y=0$ and $x=0$. One leads to $0$-limit and the other to $1$-limit. The figure below shows why:
        enter image description here






        share|cite|improve this answer









        $endgroup$















          2












          2








          2





          $begingroup$

          This limit doesn't exist. Consider two different paths $y=0$ and $x=0$. One leads to $0$-limit and the other to $1$-limit. The figure below shows why:
          enter image description here






          share|cite|improve this answer









          $endgroup$



          This limit doesn't exist. Consider two different paths $y=0$ and $x=0$. One leads to $0$-limit and the other to $1$-limit. The figure below shows why:
          enter image description here







          share|cite|improve this answer












          share|cite|improve this answer



          share|cite|improve this answer










          answered Mar 17 at 13:10









          Mostafa AyazMostafa Ayaz

          17.8k31039




          17.8k31039





















              2












              $begingroup$

              The limit does not exist: When approaching $(0,0)$ along the $x$ axis we have $|x|^y=|x|^0=1$, while when approaching along the $y$ axis we have $|x|^y=0^y=0$.






              share|cite|improve this answer











              $endgroup$

















                2












                $begingroup$

                The limit does not exist: When approaching $(0,0)$ along the $x$ axis we have $|x|^y=|x|^0=1$, while when approaching along the $y$ axis we have $|x|^y=0^y=0$.






                share|cite|improve this answer











                $endgroup$















                  2












                  2








                  2





                  $begingroup$

                  The limit does not exist: When approaching $(0,0)$ along the $x$ axis we have $|x|^y=|x|^0=1$, while when approaching along the $y$ axis we have $|x|^y=0^y=0$.






                  share|cite|improve this answer











                  $endgroup$



                  The limit does not exist: When approaching $(0,0)$ along the $x$ axis we have $|x|^y=|x|^0=1$, while when approaching along the $y$ axis we have $|x|^y=0^y=0$.







                  share|cite|improve this answer














                  share|cite|improve this answer



                  share|cite|improve this answer








                  edited Mar 17 at 13:06

























                  answered Mar 17 at 12:56









                  Barry CipraBarry Cipra

                  60.5k655128




                  60.5k655128



























                      draft saved

                      draft discarded
















































                      Thanks for contributing an answer to Mathematics Stack Exchange!


                      • Please be sure to answer the question. Provide details and share your research!

                      But avoid


                      • Asking for help, clarification, or responding to other answers.

                      • Making statements based on opinion; back them up with references or personal experience.

                      Use MathJax to format equations. MathJax reference.


                      To learn more, see our tips on writing great answers.




                      draft saved


                      draft discarded














                      StackExchange.ready(
                      function ()
                      StackExchange.openid.initPostLogin('.new-post-login', 'https%3a%2f%2fmath.stackexchange.com%2fquestions%2f3151488%2fmultivariate-limit-of-xy%23new-answer', 'question_page');

                      );

                      Post as a guest















                      Required, but never shown





















































                      Required, but never shown














                      Required, but never shown












                      Required, but never shown







                      Required, but never shown

































                      Required, but never shown














                      Required, but never shown












                      Required, but never shown







                      Required, but never shown







                      Popular posts from this blog

                      Lowndes Grove History Architecture References Navigation menu32°48′6″N 79°57′58″W / 32.80167°N 79.96611°W / 32.80167; -79.9661132°48′6″N 79°57′58″W / 32.80167°N 79.96611°W / 32.80167; -79.9661178002500"National Register Information System"Historic houses of South Carolina"Lowndes Grove""+32° 48' 6.00", −79° 57' 58.00""Lowndes Grove, Charleston County (260 St. Margaret St., Charleston)""Lowndes Grove"The Charleston ExpositionIt Happened in South Carolina"Lowndes Grove (House), Saint Margaret Street & Sixth Avenue, Charleston, Charleston County, SC(Photographs)"Plantations of the Carolina Low Countrye

                      random experiment with two different functions on unit interval Announcing the arrival of Valued Associate #679: Cesar Manara Planned maintenance scheduled April 23, 2019 at 00:00UTC (8:00pm US/Eastern)Random variable and probability space notionsRandom Walk with EdgesFinding functions where the increase over a random interval is Poisson distributedNumber of days until dayCan an observed event in fact be of zero probability?Unit random processmodels of coins and uniform distributionHow to get the number of successes given $n$ trials , probability $P$ and a random variable $X$Absorbing Markov chain in a computer. Is “almost every” turned into always convergence in computer executions?Stopped random walk is not uniformly integrable

                      How should I support this large drywall patch? Planned maintenance scheduled April 23, 2019 at 00:00UTC (8:00pm US/Eastern) Announcing the arrival of Valued Associate #679: Cesar Manara Unicorn Meta Zoo #1: Why another podcast?How do I cover large gaps in drywall?How do I keep drywall around a patch from crumbling?Can I glue a second layer of drywall?How to patch long strip on drywall?Large drywall patch: how to avoid bulging seams?Drywall Mesh Patch vs. Bulge? To remove or not to remove?How to fix this drywall job?Prep drywall before backsplashWhat's the best way to fix this horrible drywall patch job?Drywall patching using 3M Patch Plus Primer